Buy BOOKS at Discounted Price

Real Numbers

Class 10th Mathematics RS Aggarwal Solution
Exercise 1a
  1. What do you mean by Euclid’s division lemma?
  2. A number when divided by 61 gives 27 as quotient and 32 as remainder. Find the…
  3. By what number should 1365 be divided to get 31 quotient and 32 as remainder?…
  4. Using Euclid’s division algorithm, find the HCF of i. 405 and 2520 ii. 504 and…
  5. Show that every positive integer is either even or odd.
  6. Show that any positive odd integer is of the form (6m + 1) or (6m + 3) or (6m +…
  7. Show that any positive odd integer is of the form (4m + 1) or (4m + 3), where m…
Exercise 1b
  1. i. 36, 84 ii. 23, 31 iii. 96, 404 iv. 144, 198 v. 396, 1080 vi. 1152, 1664 In…
  2. i. 8, 9, 25 ii. 12, 15, 21 iii. 17, 23, 29 iv. 24, 36, 40 v. 30, 72, 432 vi. 21,…
  3. The HCF of two numbers is 23 and their LCM is 1449. If one of the numbers is…
  4. The HCF of two numbers is 145 and their LCM is 2175. If one of the numbers is…
  5. The HCF of two numbers is 18 and their product is 12960.Find their LCM.…
  6. Is it possible to have two numbers whose HCF is 18 and LCM is 760? Give reason.…
  7. Find the simplest form of: i. 69/92 ii. 473/645 iii. 1095/1168 iv. 368/496…
  8. Find the largest number which divides 438 and 606, leaving remainder 6 in each…
  9. Find the largest number which divides 320 and 457, leaving remainders 5 and 7…
  10. Find the least number which when divided by 35, 36 and 91 leaves the same…
  11. Find the smallest number which when divided by 28 and 32 leaves remainders 8…
  12. Find the smallest number which when increased by 17 is exactly divisible by…
  13. Find the greatest number of four digits which is exactly divisible by 15, 24…
  14. In a seminar, the number of participants in Hindi, English and mathematics are…
  15. Three sets of English, mathematics and science books containing 336, 240 and 96…
  16. Three pieces of timber 42 m, 49 m and 63 m long have to be divided into planks…
  17. Find the greatest possible length which can be used to measure exactly the…
  18. Find the maximum number of students among whom 1001 pens and 910 pencils can be…
  19. Find the least number of square tiles required to pave the ceiling of a room 15…
  20. Three measuring rods are 64 cm, 80 cm and 96 cm in length. Find the least…
  21. An electronic device makes a deep after every 60 seconds. Another device makes…
  22. The traffic lights at three different road crossing change after every 48…
  23. Six bells commence tolling together and toll at intervals of 2, 4, 6, 8, 10, 12…
  24. Find the missing numbers in the following factorization.
Exercise 1c
  1. Without actual division, show that each of the following rational numbers is a…
  2. Without actual division, show that each of the following rational numbers is a…
  3. Express each of the following as a fraction in simplest form. i. 0. bar 8 ii. 2.…
Exercise 1d
  1. Define (i) rational numbers
  2. Classify the following numbers as rational or irrational: i. 22/7 ii. 3.1416…
  3. Prove that each of the following numbers is irrational. i. root 6 ii. (2 - root…
  4. Prove that 1/root 3 is irrational.
  5. i. Give an example of two irrationals whose sum is rational. ii. Give an example…
  6. State whether the given statement is true or false. i. The sum of two rationals…
  7. Prove that (2 root 3-1) is an irrational numbers.
  8. Prove that (4-5 root 2) is an irrational number.
  9. Prove that (5-2 root 3) is an irrational number.
  10. Prove that 5√2 is irrational.
  11. Prove that 2/root 7 is irrational.
Exercise 1e
  1. State Euclid’s division lemma.
  2. State fundamental theorem of arithmetic.
  3. Express 360 as product of its prime factors.
  4. If a and b are two prime numbers then find HCF (a, b).
  5. If a and b are two prime numbers then LCM (a, b).
  6. If the product of two numbers is 1050 and their HCF is 25, find their LCM.…
  7. What is a composite number?
  8. If a and b are relatively prime then what is their HCF?
  9. If the rational number a/b has a terminating decimal expansion, what is the…
  10. Simplify: (2 root 45+3 root 20)/2 root 5 .
  11. Write the decimal expansion of 73/(2^4 x 5^3) .
  12. Show that there is no value of n for which (2” × 5”) ends in 5.
  13. Is it possible to have two numbers whose HCF is 25 and LCM is 520?…
  14. Give an example of two irrationals whose sum is rational.
  15. Give an example of two irrationals whose product is rational.
  16. If a and b are relatively prime, what is their LCM?
  17. The LCM of two numbers is 1200. Show that the HCF of these numbers cannot be…
  18. Express 0. bar 4 as a rational number in simplest form.
  19. Express 0. bar 23 as a rational number in simplest form.
  20. Express why 0.15015001500015… is an irrational number.
  21. Show that root 2/3 is irrational.
  22. Write a rational number between √3 and 2.
  23. Explain why 3. bar 1416 is a rational number.
Multiple Choice Questions (mcq)
  1. Which of the following is a pair of co - primes? (A) (14,35) (B) (18,25) (C) (31,93)…
  2. If a = (2^2 × 3^3 × 5^4) and b = (2^3 × 3^2 × 5) then HCF (a, b) = ?A. 90 B. 180 C. 360…
  3. HCF of (2^3 × 3^2 × 5), (2^2 × 3^3 × 5^2) and (2^4 × 3 × 5^3 × 7) isA. 30 B. 48 C. 60…
  4. LCM of (2^3 × 3 × 5) and (2^4 × 5 × 7) isA. 40 B. 560 C. 1120 D. 1680…
  5. The HCF of two numbers is 27 and their LCM is 162. If one of the numbers is 54, what is…
  6. The product of two numbers is 1600 and their HCF is 5. The LCM of the numbers isA. 8000…
  7. What is the largest number that divides each one of 1152 and 1664 exactly?A. 32 B. 64…
  8. What is the largest number that divides 70 and 125, leaving remainders 5 and 8…
  9. What is the largest number that divides 245 and 1029, leaving remainder 5 in each…
  10. The simplest form of 1095/1168 isA. 17/26 B. 25/26 C. 13/16 D. 15/16…
  11. Euclid’s division lemma states that for any positive integers a and b, there exist…
  12. A number when divided by 143 leaves 31 as remainder. What will be the remainder when…
  13. Which of the following is an irrational number?A. 22/7 B. 3.1416 C. 3. bar 1416 D.…
  14. π isA. an integer B. a rational number C. an irrational number D. none of these…
  15. 2. bar 35 isA. an integer B. a rational number C. an irrational number D. none of…
  16. 2.13113111311113… isA. an integer B. a rational number C. an irrational number D. none…
  17. The number 3.24636363… isA. an integer B. a rational number C. an irrational number D.…
  18. Which of the following rational numbers is expressible as a terminating decimal?A.…
  19. The decimal expansion of the rational number 37/2^2 x 5 will terminate afterA. one…
  20. The decimal of the number 14753/1250 will terminate afterA. one decimal place B. two…
  21. The number 1.732 isA. an irrational number B. a rational number C. an integer D. a…
  22. a and b are two positive integers such that the least prime factor of a is 3 and the…
  23. root 2 isA. a rational number B. an irrational number C. a terminating decimal D. a…
  24. 1/root 2 isA. a fraction B. a rational number C. an irrational number D. none of these…
  25. (2 + root 2) isA. an integer B. a rational number C. an irrational number D. none of…
  26. What is the least number that is divisible by all the natural numbers from 1 to 10…
Formative Assessment (unit Test)
  1. The decimal representation of 71/150 isA. a terminating decimal B. a nonterminating,…
  2. Which of the following has a terminating decimal expansion?A. 32/91 B. 19/80 C. 23/45…
  3. One dividing a positive integer n by 9, we get 7 as remainder. What will be the…
  4. Solve: 0. bar 68+0. bar 73 = ? A. 1. bar 41 B. 1. bar 42 C. 1. bar 141 D. None of these…
  5. Show that any number of the form 4n, n ∈ N can never end with the digit 0.…
  6. The HCF of two numbers is 27 and their LCM is 162. If one of the number is 81, find the…
  7. Examine whether 17/30 is a terminating decimal.
  8. First the simplest form of 148/185 .
  9. Which of the following number are irrational? a. root 2 b. cube root 6 c. 3.142857 d.…
  10. Prove that (2 +√3) is irrational.
  11. Find the HCF and LCM of 12, 15, 18, 27.
  12. Give an example of two irrationals whose sum is rational.
  13. Give prime factorization of 4620.
  14. Find the HCF of 1008 and 1080 by prime factorization method.
  15. Find the HCF and LCM of 8/9 , 10/27 and 16/81 .
  16. Find the largest number which divides 546 and 764, leaving remainders 6 and 8…
  17. Prove that root 3 is an irrational number.
  18. Show that every positive odd integer is of the form (4q + 1) or (4q + 3) for some…
  19. Show that one and only one out of n, (n + 2) and (n + 4) is divisible by 3, where n is…
  20. Show that (4+3 root 2) is irrational.

Exercise 1a
Question 1.

What do you mean by Euclid’s division lemma?


Answer:

According to Euclid’s Division Lemma if we have two positive integers a and b, then there exists unique integers q and r which satisfies the condition a = bq + r where 0≤ r ≤ b.



Question 2.

A number when divided by 61 gives 27 as quotient and 32 as remainder. Find the number.


Answer:

Let the number be x.

By Euclid’s division lemma -


x = bq + r ….(1)


where q is the quotient,


r is the remainder


and b is the divisor.


According to the question, b = 61, r = 32, q = 27.


Putting the values in equation (1) -


∴ x = 61(27) + 32


⇒ x = 1679.


Hence, the given number is 1679.



Question 3.

By what number should 1365 be divided to get 31 quotient and 32 as remainder?


Answer:

By Euclid’s division lemma -


x = bq + r ….(1)


where q is the quotient,


r is the remainder


and b is the divisor.


According to the question, x = 1365, r = 32, q = 31, b = ?.


Putting the values in equation(1) -


∴ 1365 = b(31) + 32


⇒ 31b = 1333


⇒ b = 43



Question 4.

Using Euclid’s division algorithm, find the HCF of

i. 405 and 2520

ii. 504 and 1188

iii. 960 and 1575


Answer:

i.

∵ here 405 < 2520,


∴ b = 405 and a = 2520.


By Euclid’s division lemma -


a = bq + r ….(1)


where q is the quotient, r is the remainder and b is the divisor.


Putting it in equation (1) -


⇒ 2520 = 405(6) + 90.


Here 90 is the remainder, which is not zero. Again applying the Euclid’s divison lemma -


Now, a = 405; b = 90;


⇒ 405 = 90(4) + 45.


Here 45 is the remainder, which is not zero. Again applying the Euclid’s divison lemma -


a = 90; b = 45;


⇒ 90 = 45(2) + 0.


∵ remainder is zero.


∴ HCF is 45.


ii.

∵ here 504 < 1188,


∴ b = 504 and a = 1188.


By Euclid’s division lemma -


a = bq + r ….(1)


where q is the quotient, r is the remainder and b is the divisor.


Putting the values in equation (1) -


⇒ 1188 = 504(2) + 180.


Here 180 is the remainder, which is not zero. Again applying the Euclid’s divison lemma -


Now, a = 504; b = 180;


⇒ 504 = 180(2) + 144


Here 144 is the remainder, which is not zero. Again applying the Euclid’s divison lemma -


Now,a = 180; b = 144;


⇒ 180 = 144(1) + 36.


Here 36 is the remainder, which is not zero. Again applying the Euclid’s divison lemma -


Now,a = 144; b = 36;


⇒ 144 = 36(4) + 0.


∵ remainder is zero.


∴ HCF is 36.


iii.

∵ here 960 < 1575,


∴ b = 960 and a = 1575.


a = bq + r ….(1)


where q is the quotient, r is the remainder and b is the divisor.


Putting the values in equation (1) -


⇒ 1575 = 960(1) + 615.


Here 615 is the remainder, which is not zero. Again applying the Euclid’s divison lemma -


Now, a = 960; b = 615;


⇒ 960 = 615(1) + 345.


Here 345 is the remainder, which is not zero. Again applying the Euclid’s divison lemma -


Now, a = 615; b = 345;


⇒ 615 = 345(1) + 270.


Here 270 is the remainder, which is not zero. Again applying the Euclid’s divison lemma -


Now, a = 345; b = 270;


⇒ 345 = 270(1) + 75.


Here 75 is the remainder, which is not zero. Again applying the Euclid’s divison lemma -


Now, a = 270; b = 75;


⇒ 270 = 75(3) + 45.


Here 45 is the remainder, which is not zero. Again applying the Euclid’s divison lemma -


Now, a = 75; b = 45;


⇒ 75 = 45(1) + 30.


Here 30 is the remainder, which is not zero. Again applying the Euclid’s divison lemma -


Now, a = 45; b = 30;


⇒ 45 = 30(1) + 15.


Here 15 is the remainder, which is not zero. Again applying the Euclid’s divison lemma -


a = 30; b = 15;


⇒ 30 = 15(2) + 0.


∵ remainder is zero.


∴ HCF is 15.


Question 5.

Show that every positive integer is either even or odd.


Answer:

Using Euclid’s divison lemma -

a = bq + r;


We want to show that any even integer is of the form '2q' and any odd integer is of the form'2q + 1'


So, we take the two integers a and 2.


When we divide a with 2, the possible values of remainder are 0 and 1, which means


a = 2q + 0 or a = 2q + 1


Also, 2q is always an even integer for any integral value of q and so 2q + 1 will always be an odd integer. (∵ 'even integer + 1' is always an odd integer)


∴ when a is a positive even integer it will always be of the form 2q


And when a is positive odd integer it will always be of the form 2q + 1.


∴ for every positive integer it is either odd or even.



Question 6.

Show that any positive odd integer is of the form (6m + 1) or (6m + 3) or (6m + 5), where m is some integer.


Answer:

Let take a as any positive integer and b = 6. a > b

Then using Euclid’s algorithm, we get a = 6q + r here r is remainder and value of q is more than or equal to 0 and r = 0, 1, 2, 3, 4, 5 because 0 ≤ r < b and the value of b is 6


So total possible forms will be 6q + 0, 6q + 1 , 6q + 2,6q + 3,6q + 4,6q + 5.


6q + 0 → 6 is divisible by 2 so it is an even number.


6q + 1 → 6 is divisible by 2 but 1 is not divisible by 2 so it is an odd number.


6q + 2 → 6 is divisible by 2 and 2 is also divisible by 2 so it is an even number.


6q + 3 → 6 is divisible by 2 but 3 is not divisible by 2 so it is an odd number.


6q + 4 → 6 is divisible by 2 and 4 is also divisible by 2 it is an even number.


6q + 5 → 6 is divisible by 2 but 5 is not divisible by 2 so it is an odd number.


So odd numbers will in form of 6q + 1, or 6q + 3, or 6q + 5.



Question 7.

Show that any positive odd integer is of the form (4m + 1) or (4m + 3), where m is some integer.


Answer:

Let a be any odd positive integer and b = 4. By division lemma there exist integer q and r such that

a = 4 q + r, where 0 ≤ r ≤ 4


so a = 4q or, a = 4q + 1 or, a = 4q + 2 or, a = 4q + 3


4q + 1 → 4 is divisible by 2 but 1 is not divisible by 2, so it is an odd number


4q + 2 → 4 is divisible by 2 and 2 is also divisible by 2, so it is an even number


4q + 3 → 4 is divisible by 2 but 3 is not divisible by 2, so it is an odd number


4q + 4 → 4 is divisible by 2 and 4 is also divisible by 2, so it is an even number


∴ any odd integer is of the form 4q + 1 or, 4q + 3.




Exercise 1b
Question 1.

Using prime factorization, find the HCF and LCM of:

i. 36, 84 ii. 23, 31

iii. 96, 404 iv. 144, 198

v. 396, 1080 vi. 1152, 1664

In each case, verify that:

HCF x LCM = product of given numbers.


Answer:

i. HCF = 12, LCM = 252


Prime factorization of given numbers -


36 = 2 × 2 × 3 × 3


84 = 2 × 2 × 3 × 7


So HCF = product of common factors = 2 × 2 × 3 = 12


And LCM = product of prime factors with highest powers = 22 × 32 × 7 = 252


Verification -


HCF × LCM = product of numbers


⇒ LHS = 12 × 252 = 3024


⇒ RHS = 36 × 84 = 3024


∵ LHS = RHS(Hence verified)


ii. HCF = 1, LCM = 713


Prime factorization of given numbers -


23 = 23 × 1


31 = 31 × 1


So HCF = product of common factors = 1


And LCM = product of prime factors with highest powers = 23 × 31 = 713


Verification -


HCF × LCM = product of numbers


⇒ LHS = 1 × 713 = 713


⇒ RHS = 23 × 31 = 713


∵ LHS = RHS(Hence verified)


iii. HCF = 4, LCM = 9696


Prime factorization of given numbers -


96 = 2 × 2 × 2 × 2 × 2 × 3


404 = 2 × 2 × 101


So HCF = product of common factors = 2 × 2 = 4


And LCM = product of prime factors with highest powers = 25 × 3 × 101 = 9696.


Verification -


HCF × LCM = product of numbers


⇒ LHS = 4 × 9696 = 38784


⇒ RHS = 96 × 404 = 38784


∵ LHS = RHS(Hence verified)


iv. HCF = 18, LCM = 1584


Prime factorization of given numbers -


144 = 2 × 2 × 2 × 2 × 3 × 3


198 = 2 × 3 × 3 × 11


So HCF = product of common factors = 2 × 3 × 3 = 18


And LCM = product of prime factors with highest powers = 24 × 32 × 11 = 1584


Verification -


HCF × LCM = product of numbers


⇒ LHS = 18 × 1584 = 28512


⇒ RHS = 144 × 198 = 28512


∵ LHS = RHS(Hence verified)


v. HCF = 36, LCM = 11880


Prime factorization of given numbers -


396 = 2 × 2 × 3 × 3 × 11


1080 = 2 × 2 × 2 × 3 × 3 × 3 × 5


So HCF = product of common factors = 2 × 2 × 3 × 3 = 36


And LCM = product of prime factors with highest powers = 23 × 33 × 5 × 11 = 11880


Verification -


HCF × LCM = product of numbers


⇒ LHS = 36 × 11880 = 427680


⇒ RHS = 396 × 1080 = 427680


∵ LHS = RHS(Hence verified)


vi. HCF = 128, LCM = 14976


Prime factorization of given numbers -


1152 = 2 × 2 × 2 × 2 × 2 × 2 × 2 × 3 × 3


1664 = 2 × 2 × 2 × 2 × 2 × 2 × 2 × 13


So HCF = product of common factors = 2 × 2 × 2 × 2 × 2 × 2 × 2 = 128


And LCM = product of prime factors with highest powers = 27 × 32 × 13 = 14976


Verification -


HCF × LCM = product of numbers


⇒ LHS = 128 × 14976 = 1916928


⇒ RHS = 1152 × 1664 = 1916928


∵ LHS = RHS(Hence verified)



Question 2.

Using prime factorization, find the HCF and LCM of:

i. 8, 9, 25 ii. 12, 15, 21

iii. 17, 23, 29 iv. 24, 36, 40

v. 30, 72, 432 vi. 21, 28, 36, 45


Answer:

i. HCF = 1, LCM = 1800


Prime factorization of given numbers -


8 = 2 × 2 × 2 × 1


9 = 3 × 3 × 1


25 = 5 × 5 × 1


So HCF = product of common factors = 1


And LCM = product of prime factors with highest powers = 23 × 32 × 52 = 1800


ii. HCF = 3, LCM = 420


Prime factorization of given numbers -


12 = 2 × 2 × 3


15 = 3 × 5


21 = 3 × 7


So HCF = product of common factors = 3


And LCM = product of prime factors with highest powers = 22 × 3 × 5 × 7 = 420


iii. HCF = 1, LCM = 11339


Prime factorization of given numbers -


17 = 17 × 1


23 = 23 × 1


29 = 29 × 1


So HCF = 1


And LCM = product of prime factors with highest powers = 17 × 23 × 29 = 11339


iv. HCF = 4, LCM = 360


Prime factorization of given numbers -


36 = 2 × 2 × 3 × 3


24 = 2 × 2 × 2 × 3


40 = 2 × 2 × 2 × 5


So HCF = product of common factors = 2 × 2 = 4


And LCM = product of prime factors with highest powers = 23 × 32 × 5 = 360


v. HCF = 6, LCM = 2160


Prime factorization of given numbers -


30 = 2 × 3 × 5


72 = 2 × 2 × 2 × 3 × 3


432 = 2 × 2 × 2 × 2 × 3 × 3 × 3


So HCF = product of common factors = 2 × 3 = 6


And LCM = product of prime factors with highest powers = 24 × 33 × 5 = 2160


vi. HCF = 1, LCM = 1260


Prime factorization of given numbers -


21 = 3 × 7


28 = 2 × 2 × 7


36 = 2 × 2 × 3 × 3


45 = 3 × 3 × 5


So HCF = product of common factors = 1


And LCM = product of prime factors with highest powers = 22 × 32 × 5 × 7 = 1260



Question 3.

The HCF of two numbers is 23 and their LCM is 1449. If one of the numbers is 161, find the other.


Answer:

We know that LCM × HCF = product of numbers.

Let the other number be x.


So, 161 × x = 23 × 1449


⇒ x = 207



Question 4.

The HCF of two numbers is 145 and their LCM is 2175. If one of the numbers is 725, find the other.


Answer:

We know that LCM × HCF = product of numbers.


Let the other number be x.


So, 725 × x = 145 × 2175


⇒ x = 435



Question 5.

The HCF of two numbers is 18 and their product is 12960.Find their LCM.


Answer:

We know that LCM × HCF = product of numbers.


Let the LCM of numbers to be x.


So, 18 × x = 12960


⇒ x = 720



Question 6.

Is it possible to have two numbers whose HCF is 18 and LCM is 760? Give reason.


Answer:

No, ∵ HCF does not divided LCM exactly


Using Euclid’s division lemma -


Take a = 760 and b = 18.


a = bq + r. where q is the quotient, r is the remainder and b is the divisor.


If HCF divides LCM completely, r = 0.


Putting the values -


Here 760 = 18(42) + 4


⇒ r = 4


∵ r is not equal to zero


∴ HCF does not divides LCM completely.


So this is not possible for two numbers to have HCF = 18 and LCM = 760.



Question 7.

Find the simplest form of:

i. ii.

iii. iv.


Answer:

(i) HCF for 69 and 92 is 23


So dividing both of them by 23 -


69 = 23 × 3


92 = 23 × 4


We get numerator as 3 and denominator as 4.


∴ simplest fraction is .


(ii) HCF for 473 and 645 is 43


So dividing both of them by 43 -


473 = 43 × 11


645 = 43 × 15


We get numerator as 11 and denominator as 15


∴ simplest fraction is .


(iii) HCF for 368 and 496 is 16


So, dividing both of them by 16 -


368 = 16 × 23


496 = 16 × 31


We get numerator as 23 and denominator as 31.


∴ simplest fraction is


(iv) HCF for 1095 and 1168 is 73


So, dividing both of them by 73 -


1095 = 73 × 15


1168 = 73 × 16


We get numerator as 15 and denominator as 16.


∴ simplest fraction is .


Question 8.

Find the largest number which divides 438 and 606, leaving remainder 6 in each case.


Answer:

∵ we are getting 6 as remainder.

Subtracting 6 from the dividend will give us the exact division.


So, let’s subtract the remainder and find the HCF of numbers (largest number which divides both the number)


We need to find HCF of 438 - 6 = 432 and 606 - 6 = 600.


Prime factorization of numbers -


432 = 2 × 2 × 2 × 2 × 3 × 3 × 3


600 = 2 × 2 × 2 × 3 × 5 × 5


HCF of given numbers = 2 × 2 × 2 × 3 = 24.



Question 9.

Find the largest number which divides 320 and 457, leaving remainders 5 and 7 respectively.


Answer:

∵ we are getting 5 and 7 as remainder.


Subtracting 5 and 7 from the dividends will give us the exact division.


So, let’s subtract the remainder and find the HCF of numbers (largest number which divides both the number) -


We need to find HCF of 320 - 5 = 315 and 457 - 7 = 450.


Prime factorization of numbers -


315 = 3 × 3 × 5 × 7


450 = 2 × 3 × 3 × 5 × 5


HCF of given numbers = 3 × 3 × 5 = 45.



Question 10.

Find the least number which when divided by 35, 56 and 91 leaves the same remainder 7 in each case.


Answer:

Find the LCM of the numbers (the least number which all the given numbers divides) -


Prime factorization of numbers -


35 = 5 × 7


56 = 2 × 2 × 2 × 7


91 = 7 × 13


LCM of given numbers = product of prime factors with highest powers = 23 × 5 × 7 × 13 = 3640.


∵ required remainder is 7.


We need to add 7 to the LCM of numbers.


∴ the least number that leaves remainder 7 will be 3640 + 7 = 3647.


Question 11.

Find the smallest number which when divided by 28 and 32 leaves remainders 8 and 12 respectively.


Answer:

Subtract the remainders from the given numbers:


If you look at the negative remainders -
Remainder from 28 = 28 - 8 is - 20.
Remainder from 32 = 32 - 12 is - 20


Find the LCM of the numbers (the least number which all the given numbers divide)


Prime factorization of numbers -


28 = 2 × 2 × 7


32 = 2 × 2 × 2 × 2 × 2


LCM of given numbers = product of prime factors with highest powers = 25 × 7 = 224.


Smallest no, which leaves remainder 8 and 12 when divided by 28 and 32


= LCM - 20 = 224 - 20 = 204



Question 12.

Find the smallest number which when increased by 17 is exactly divisible by both 468 and 520.


Answer:

Find the LCM of the numbers (the least number which all the given numbers divide) -


Prime factorization of numbers -


468 = 2 × 2 × 3 × 3 × 13


520 = 2 × 2 × 2 × 5 × 13


LCM of given numbers = product of prime factors with highest powers = 23 × 32 × 5 × 13 = 4680


∵ the number is increased by 17 to get perfect division.


We need to subtract 17 to the LCM of numbers.


∴ the least number when added by 17 gives exact division will be 4680 - 17 = 4663.


Question 13.

Find the greatest number of four digits which is exactly divisible by 15, 24 and 36.


Answer:

Find the LCM of the numbers (the least number which all the given numbers divide) -


Prime factorization of numbers -


15 = 3 × 5


24 = 2 × 2 × 2 × 3


36 = 2 × 2 × 3 × 3


LCM of given numbers = product of prime factors with highest powers = 23 × 32 × 5

= 360


We know that the greatest four-digit number is 9999.


So, the greatest value closest to 9999 which 360 can divide can be found by Euclid’s division lemma -


Putting the values in the equation a = bq + r -


9999 = 360(27) + 279


So, highest four - digit number 360 can completely divide = 9999 - remainder

= 9999 - 279

= 9720.


Question 14.

In a seminar, the number of participants in Hindi, English and mathematics are 60, 84 and 108 respectively. Find the minimum number of rooms required, if in each room, the same numbers of participants are to be seated and all of them being in the same subject.


Answer:

The number of room will be minimum if each room accommodates maximum number of participants.

∵ in each room the same number of participants are to be seated and all of them must be of the same subject.


Therefore, the number of participants in each room must be the HCF of 60, 84 and 108


So HCF of 60, 84 and 108 -


Prime factors of numbers are -


60 = 2 × 2 × 3 × 5


84 = 2 × 2 × 3 × 7


108 = 2 × 2 × 3 × 3 × 3


∴ HCF = 2 × 2 × 3 = 12


Therefore, in each room 12 participants can be seated.


Total number of students = 60 + 84 + 108 = 252


So minimum number of room required to accommodate all students = 252/12 = 21



Question 15.

Three sets of English, mathematics and science books containing 336, 240 and 96 books respectively have to be stacked in such a way that all the books are stored subject wise and the height of each stack is the same. How many stacks will be there?


Answer:

The number of stacks will be minimum if each stack accommodates maximum number of books.


∵ height of each stack is the same and all of them are of the same subject.


Therefore, the number of books in each stack must be the HCF of 336,240 and 96.


So HCF of 336,240 and 96 -


Prime factors of numbers are -


336 = 2 × 2 × 2 × 2 × 3 × 7


240 = 2 × 2 × 2 × 2 × 3 × 5


96 = 2 × 2 × 2 × 2 × 2 × 3


∴ HCF = 2 × 2 × 2 × 2 × 3 = 48


Therefore, in each stack 48 books can be placed.


Total number of books = 336 + 240 + 96 = 672


So minimum number of stacks required to accommodate all books = 672/48 = 14



Question 16.

Three pieces of timber 42 m, 49 m and 63 m long have to be divided into planks of the same length. What is the greatest possible length of each plank? How many planks are formed?


Answer:

∵ length of each plank is the same and we are supposed to find plank of greatest possible length.


Therefore, the greatest possible length of each plank must be the HCF of 42, 49 and 63.


So HCF of 42, 49 and 63 -


Prime factors of numbers are -


42 = 2 × 3 × 7


49 = 7 × 7


63 = 3 × 3 × 7


∴ HCF = 7


Therefore, maximum plank length of each plank = 7m


Total available length of plank = 42 + 49 + 63 = 154m


So number of planks of this maximum possible length = 154/7 = 22.



Question 17.

Find the greatest possible length which can be used to measure exactly the lengths 7 m, 3 m 85 cm and 12 m 95 cm.


Answer:

We know 1m = 100cm.


So,


7m = 700cm,


3m 85cm = 300 + 85 = 385cm,


and 12m 95cm = 1200 + 95 = 1295cm.


The greatest possible length that can measure all the three given lengths will be HCF of the lengths -


So HCF of 700, 385 and 1295 -


Prime factors of numbers are -


700 = 2 × 2 × 5 × 5 × 7


385 = 5 × 7 × 11


1295 = 5 × 7 × 37


∴ HCF = 5 × 7 = 35.


So the greatest possible length that can be used = 35cm.



Question 18.

Find the maximum number of students among whom 1001 pens and 910 pencils can be distributed in such a way that each student gets the same number of pens and the same number of pencils.


Answer:

∵ each student gets the same number of pens and pencils.

Maximum number of students among which it can be distributed will be equal to HCF of number of pencils and number of pens.


So HCF of 1001 and 910 -


Prime factors of the numbers are -


1001 = 7 × 11 × 13


910 = 2 × 5 × 7 × 13


∴ HCF = 7 × 13 = 91.



Question 19.

Find the least number of square tiles required to pave the ceiling of a room 15 m 17 cm long and 9 m 2 cm broad.


Answer:

We know 1m = 100cm.


So,


15m 17cm = 1500 + 17 = 1517cm


And 9m 2cm = 900 + 2 = 902 cm


Least number of square tiles required to pave the ceiling of this area if largest tiles are used.


Length of largest tile = H.C.F. of 1517 cm and 902 cm


Prime factors of 1517 and 902 -


1517 = 41 × 37


902 = 2 × 11 × 41


So HCF = 41


∴ Area of tiles of largest length and width = (41 × 41) cm2


And area of room = length × width = (1517 × 902) cm2


So minimum number of tiles required = (1517 × 902) cm2/ (41 × 41) cm2 = 814.



Question 20.

Three measuring rods are 64 cm, 80 cm and 96 cm in length. Find the least length of cloth that can be measured an exact number of times, using any of the rods


Answer:

Least length of cloth that can be measured using this rod will be LCM of the three lengths.

So LCM of 64, 80 and 96 -


Prime factors of the given numbers are -


64 = 2 × 2 × 2 × 2 × 2 × 2


80 = 2 × 2 × 2 × 2 × 5


96 = 2 × 2 × 2 × 2 × 2 × 3


So LCM = product of prime factors with highest powers = 26 × 3 × 5 = 960


∴ least length of cloth that can be measured using the rods = 960 cm = 9.6m



Question 21.

An electronic device makes a deep after every 60 seconds. Another device makes a beep after every 62 seconds. They beeped together at 10 a.m. At what time will they beep together at the earliest?


Answer:

The devices will beep simultaneously at the LCM of intervals of beeps

So LCM of 60 and 62 -


Prime factors of the numbers are -


60 = 2 × 2 × 3 × 5


62 = 2 × 31


So LCM = product of prime factors with highest powers = 22 × 3 × 5 × 31 = 1860 seconds


∴ devices will beep simultaneously after 1860 seconds.


We know 1 min = 60 seconds,


So 1860 seconds = 1860/60 minutes = 31 minutes.


∴ the next time they will beep simultaneously at 10:31 hrs.



Question 22.

The traffic lights at three different road crossing change after every 48 seconds, 72 seconds and 108 seconds respectively. If they all change simultaneously at 8 a.m., then at what time will they again change simultaneously?


Answer:

The traffic lights will change at the LCM of intervals of all the three lights.

So LCM of 48, 72 and 108 -


Prime factors of the numbers are -


48 = 2 × 2 × 2 × 2 × 3


72 = 2 × 2 × 2 × 3 × 3


108 = 2 × 2 × 3 × 3 × 3


So LCM = product of prime factors with highest powers = 24 × 33 = 432


∴ traffic light will change after 432 seconds.


We know 1 min = 60 seconds,


So 432 seconds = 432/60 minutes = 7 minutes and 12 seconds.


∴ the next time lights will simultaneously change at 8:7:12 hrs.



Question 23.

Six bells commence tolling together and toll at intervals of 2, 4, 6, 8, 10, 12 minutes respectively. In 30 hours, how many times do they toll together?


Answer:

All the clocks will toll together at the LCM of their intervals.

So LCM of the intervals = 2,4,6,8,10,12 = 120.


∴ all clocks will toll together after every 120 minutes.


Given interval = 30 hrs = 30 × 60 minutes = 1800 minutes {∵ 1hr = 60 minutes}


∴ number of times clock will tol = 1800/120 = 15.


plus one time when they commenced together


So the answer will be 16 times.



Question 24.

Find the missing numbers in the following factorization.



Answer:

Let’s start from the bottom -

The factors of last numbers are 11 and 5.


So last number is 11 × 5 = 55


Now factors of the second number from bottom is 55 and 3


So second last number is 55 × 3 = 165


Also factors for the third number from bottom is 165 and 2.


So third last number is 165 × 2 = 330


Similarly factors for the first number is 330 and 2.


So first number is 330 × 2 = 660




Exercise 1c
Question 1.

Without actual division, show that each of the following rational numbers is a terminating decimal. Express each in decimal form.

i. ii.

iii. iv.

v. vi.


Answer:

(i)


∵ denominator is of the form, 2n × 5m,


where n = 3 and m = 2.


∴ it is terminating in nature.



ii.


∵ denominator is of the form, 2n × 5m,


125 = 5 × 5 × 5.


where n = 0 and m = 3.


∴ it is terminating in nature.



iii.


∵ denominator is of the form, 2n × 5m,


800 = 2 × 2 × 2 × 2 × 2 × 2 × 5 × 5


where n = 5 and m = 2.


∴ it is terminating in nature.



iv.


∵ denominator is of the form, 2n × 5m,


1600 = 2 × 2 × 2 × 2 × 2 × 2 × 2 × 5 × 5


where n = 6 and m = 2.


∴ it is terminating in nature.



v.


∵ denominator is of the form, 2n × 5m,


320 = 2 × 2 × 2 × 2 × 2 × 2 × 2 × 5


where n = 6 and m = 1.


∴ it is terminating in nature.



vi.


∵ denominator is of the form, 2n × 5m,


3125 = 5 × 5 × 5 × 5 × 5


where n = 0 and m = 5.


∴ it is terminating in nature.




Question 2.

Without actual division, show that each of the following rational numbers is a non-terminating repeating decimal.

i. ii.

iii. iv.

v. vi.

vii. viii.


Answer:

(i) Denominator is 23 × 3 which is not in the form 2n × 5m

∴ the fraction will not be a terminating decimal.


(ii) Denominator is 22 × 33 × 5 which is not in the form 2n × 5m


∴ the fraction will not be a terminating decimal.


(iii) Denominator is 22 × 53 × 72 which is not in the form 2n × 5m


∴ the fraction will not be a terminating decimal.


(iv) Denominator is 35 = 5 × 7 which is not in the form 2n × 5m


∴ the fraction will not be a terminating decimal.


(v) Denominator is 210 = 2 × 3 × 5 × 7 which is not in the form 2n × 5m


∴ the fraction will not be a terminating decimal.


(vi) Denominator is 147 = 3 × 7 × 7 which is not in the form 2n × 5m


∴ the fraction will not be a terminating decimal.


(vii) Denominator is 343 = 7 × 7 × 7 which is not in the form 2n × 5m


∴ the fraction will not be a terminating decimal.


(viii) Denominator is 455 = 5 × 91, which is not in the form 2n × 5m


∴ the fraction will not be a terminating decimal.



Question 3.

Express each of the following as a fraction in simplest form.

i. ii.

iii. iv.

v. vi.


Answer:

i) Let x = 0. - (i)


Multiply 10 on both sides -


10x = 8. - (ii)


Subtract the equations (i) from (ii) -


9x = 8


So x =


ii) Let x = 2. - (i)


Multiply 10 on both sides -


10x = 24. - (ii)


Subtract the equations (i) from (ii) -


9x = 22


So x =


iii) Let x = 0. - (i)


Multiply 100 on both sides -


100x = 24. - (ii)


Subtract the equations (i) from (ii) -


99x = 24


So x = =


iv) Let x = 0.1 - (i)


Multiply 10 on both sides -


10x = 1. - (ii)


Multiply 10 on both sides -


100x = 12. - (iii)


Subtract the equations (ii) from (iii) -


90x = 11


So x =


v) Let x = 2.2 - (i)


Multiply 10 on both sides -


10x = 22. - (ii)


Multiply 10 on both sides -


100x = 224. - (iii)


Subtract the equations (ii) from (iii) -


90x = 202


So x = =


vi) Let x = 0. - (i)


Multiply 1000 on both sides -


1000x = 365. - (ii)


Subtract the equations (i) from (ii) -


999x = 365


So x =



Exercise 1d
Question 1.

Define (i) rational numbers


Answer:

A rational number is any number that can be expressed as the quotient or fraction p/q of two integers, a numerator p and a non - zero denominator q.


(ii) irrational numbers


An irrational number is a number that cannot be expressed as a fraction p/q for any integers p and q. Irrational numbers have decimal expansions that neither terminate nor become periodic


(iii) real numbers.


Real numbers are numbers that can be found on the number line. This includes both the rational and irrational numbers.



Question 2.

Classify the following numbers as rational or irrational:

i. ii. 3.1416

iii. π iv.

v. 5.636363 … vi. 2.040040004 …

vii. 1.535335333 … viii. 3.121221222 …

ix.


Answer:

A rational number is any number that can be expressed as the quotient or fraction p/q of two integers, a numerator p and a non - zero denominator q. While, an irrational number is a number that cannot be expressed as a fraction p/q for any integers p and q. Irrational numbers have decimal expansions that neither terminate nor become periodic.

{π,3., 5.636363 …, 2.040040004 …, , 1.535335333 …, 3.121221222 …, are irrational}


i. rational ii. rational


iii. irrational iv. rational


v. rational vi. irrational


vii. irrational viii. irrational


ix. irrational x. irrational


Question 3.

Prove that each of the following numbers is irrational.

i. ii.

iii. iv.

v. vi.

vii. viii.

xi.


Answer:

(i) let's assume that √6 is rational.

By definition, that means there are two integers a and b with no common divisors where:


⇒ a/b = √6


So let's take square of both the sides -


⇒ (a/b)(a/b) = (√6)( √6)
⇒ a2/b2 = 6
⇒ a2 = 6b2


Last statement means the RHS (right hand side) is even, because it is a product of integers and one of those integers (at least 6) is even. So a2 must be even. But any odd number times itself is odd, so if a2 is even, then a is even.


Since a is even, there is some integer c such that:


⇒ 2c = a.


Now let's replace a with 2c:


⇒ a2 = 6b2
⇒ (2c)2 = (2)(3)b2
⇒ 2c2 = 3b2


But now we can argue the same thing for b, because the LHS is even, so the RHS must be even and that means b is even.


Now this is the contradiction: if a is even and b is even, then they have a common divisor (2). Then our initial assumption must be false, so the square root of 6 cannot be rational.


Thus, proved that √6 is irrational.


(ii) assume that 2 - √3 is rational
2 - √3 = a/b , where a and b are integers .
⇒ - √3 = a/b - 2
⇒ √3 = 2 - a/b
⇒ √3 = 2b/b - a/b
⇒ √3 = (2b – a) / b
we know that a, b and 2 are integers and they are also rational {i.e RHS is rational}
therefore √3 will be rational.
but we know that √3 is irrational.
there is a contradiction
so, 2 - √3 is an irrational number


(iii) assume that 3 + √2 is rational
3 + √2 = a/b , where a and b are integers .
⇒ √2 = a/b - 3
⇒ √2 = (a - 3b)/b
we know that a, b and 3 are integers and they are also rational {i.e RHS is rational}
therefore √2 will be rational.
but we know that √2 is irrational.
there is a contradiction
so, 3 + √2 is an irrational number


(iv) assume that 2 + √5 is rational
2 + √5 = a/b , where a and b are integers .
⇒ √5 = a/b - 2
⇒ √5 = a/b - 2b/b
⇒ √5 = (a - 2b)/ b
we know that a, b and 2 are integers and they are also rational {i.e RHS is rational}
therefore √5 will be rational.
but we know that √5 is irrational.
there is a contradiction
so, 2 + √5 is an irrational number


(v) assume that 5 + 3√2 is rational
5 + 3√2 = a/b , where a and b are integers .
⇒ 3√2 = a/b - 5
⇒ 3√2 = a/b - 5b/b
⇒ √2 = (a - 5b)/3b
we know that a, b, 3 and 5 are integers and they are also rational {i.e RHS is rational}
therefore √2 will be rational.
but we know that √2 is irrational.
there is a contradiction
so, 5 + 3√2 is an irrational number.


(vi) assume that 3√7 is rational
⇒ 3√7 = a/b , where a and b are integers .
⇒ √7 = a/3b
we know that a, b and 3 are integers and they are also rational {i.e RHS is rational}
therefore √7 will be rational.
but we know that √7 is irrational.
there is a contradiction
so, 3√7 is an irrational number


(vii) {Rationalising}


Now, let us assume that 3√5 / 5 is a rational number


⇒ 3√5/5 = p/q (where, p & q are integers, q not equal to 0)


⇒ 3√5 = 5p/q


⇒ √5 = 5p/3q


Here, LHS is an irrational number whereas RHS is a rational number.


So by contradiction 3√5 / 5 is an irrational number


(viii) assume that 2 - 3√5 is rational
2 - 3√5 = a/b , where a and b are integers .
⇒ - 3√5 = a/b - 2
⇒ 3√5 = 2 - a/b
⇒ 3√5 = 2b/b - a/b
⇒ √5 = (2b – a) / 3b
we know that a, b, 2 and 3 are integers and they are also rational {i.e RHS is rational}
therefore √5 will be rational.
but we know that √5 is irrational.
there is a contradiction
so, 2 - 3√5 is an irrational number


(ix) Multiplying both sides by (√5 - √3).
(√5 - √3) (√5 + √3) = 5 - 3 = 2


⇒ 2 = p/q × (√5 - √3)
⇒ (√5 - √3) = 2q/p,


∴ √5 - √3 is rational = 2q/p
⇒ √5 + √3 = p/q
⇒ √5 - √3 = 2q/p


Adding the equations -


, which is a rational number.


But we know that √5 is IRRATIONAL.


Therefore, the assumption is wrong and √3 + √5 is irrational.


Question 4.

Prove that is irrational.


Answer:

Assume to be rational. So we can write it in the form of a/b where a and b are co - prime.

So , a/b =


And ∴ b/a = √3


Since b/a is rational but √3 is irrational.


By contradiction is irrational.


.



Question 5.

i. Give an example of two irrationals whose sum is rational.

ii. Give an example of two irrational whose product is rational.


Answer:

Let’s take (2 + ) and (2 ‒ ).

These are irrational numbers.


Their sum = (2 + ) + (2 ‒ ) = 4 {which is rational}.


ii) Let’s take (3 + ) and (3 ‒ ).


These are irrational numbers.


Their product = (3 + ) × (3 ‒ ) = 9 + 3√2 – 3√2 – (√2)2


= 9 – 4 = 5 {which is rational}



Question 6.

State whether the given statement is true or false.

i. The sum of two rationals is always rational.

ii. The product of two rationals is always rational.

iii. The sum of two irrationals is always an irrational.

iv. The product of two irrationals is always an irrational.

v. The sum of rational and an irrational is irrational.

vi. The product of a rational and an irrational is irrational.


Answer:

i. True ii. True


iii. False iv. False


v. True vi. True


(iii) Let’s take (2 + ) and (2 ‒ ).


These are irrational numbers.


Their sum = (2 + ) + (2 ‒ ) = 4 {which is rational}.


∴ it is false.


(iv) Let’s take (3 + ) and (3 ‒ ).


These are irrational numbers.


Their product = (3 + ) × (3 ‒ ) = 9 + 3√2 – 3√2 – (√2)2


= 9 – 4 = 5 {which is rational}


∴ it is false.



Question 7.

Prove that is an irrational numbers.


Answer:

Assume that 2√3 - 1 is rational
2√3 = a/b , where a and b are integers .
⇒ 2√3 = a/b + 1
⇒ 2√3 = a/b + b/b
⇒ √3 = (a + b) / 2b
we know that a, b, and 2 are integers and they are also rational {i.e RHS is rational}
therefore √3 will be rational.
but we know that √3 is irrational.
there is a contradiction
so, 2√3 - 1 is an irrational number.



Question 8.

Prove that is an irrational number.


Answer:

5√2 is an irrational number and subtraction of a rational number and an irrational number is an irrational number.

∴, 4 - 5√2 is irrational, where 5√2 is an irrational number while 4 is a rational number.


Alternative - Assume that 4 - 5√2 is rational
4 - 5√2 = a/b , where a and b are integers .
⇒ - 5√2 = a/b - 4
⇒ 5√2 = 4 - a/b
⇒ 5√2 = 4b/b - a/b
⇒ √2 = (4b – a) / 5b
we know that a, b, 4 and 5 are integers and they are also rational {i.e RHS is rational}
therefore √2 will be rational.
but we know that √2 is irrational.
there is a contradiction
so, 4 - 5√2 is an irrational number



Question 9.

Prove that is an irrational number.


Answer:

2√3 is an irrational number and subtraction of a rational number and an irrational number is an irrational number.

∴, 5 - 2√3 is irrational, where 2√3 is an irrational number while 5 is a rational number.


Alternative - Assume that 5 - 2√3 is rational
5 - 2√3 = a/b , where a and b are integers .
⇒ - 2√3 = a/b - 5
⇒ 2√3 = 5 - a/b
⇒ 2√3 = 5b/b - a/b
⇒ √3 = (5b – a) / 2b
we know that a, b, 2 and 5 are integers and they are also rational {i.e RHS is rational}
therefore √3 will be rational.
but we know that √3 is irrational.
there is a contradiction
so, 5 - 2√3 is an irrational number



Question 10.

Prove that 5√2 is irrational.


Answer:

assume that 5√2 is rational

where a and b are integers .

we know that a, b and 5 are integers and they are also rational {i.e RHS is rational}
therefore √2 will be rational.
but we know that √2 is irrational.
there is a contradiction
so, 5√2 is an irrational number

∴ it is proved that 5√2 is irrational.


Question 11.

Prove that is irrational.


Answer:


Now, let us assume that 2√7 / 7 is a rational number


⇒ 2√7/7 = p/q (where, p & q are integers, q not equal to 0)


⇒ 2√7 = 7p/q


⇒ √7 = 7p/2q


Here, LHS is an irrational number whereas RHS is a rational number.


So by contradiction 2√7 / 7 is an irrational number




Exercise 1e
Question 1.

State Euclid’s division lemma.


Answer:

According to Euclid’s Division Lemma if we have two positive integers a and b, then there exists unique integers q and r which satisfies the condition a = bq + r where 0≤ r ≤ b.



Question 2.

State fundamental theorem of arithmetic.


Answer:

The fundamental theorem of arithmetic, also called the unique factorization theorem or the unique - prime - factorization theorem, states that every integer greater than 1 either is prime itself or is the product of prime numbers, and that this product is unique, up to the order of the factors.



Question 3.

Express 360 as product of its prime factors.


Answer:

Factorization of 360 = 2 × 2 × 2 × 3 × 3 × 5 = (23× 32× 5)



Question 4.

If a and b are two prime numbers then find HCF (a, b).


Answer:

Two prime numbers don’t have a common factor other than 1. So HCF = 1.



Question 5.

If a and b are two prime numbers then LCM (a, b).


Answer:

∵ HCF of two prime numbers is 1 and HCF × LCF = product of numbers.

So LCM of numbers = product of numbers = a × b = ab.



Question 6.

If the product of two numbers is 1050 and their HCF is 25, find their LCM.


Answer:

∵ HCF × LCM = product of numbers

∴ 25 × LCM = 1050


And LCM = 1050/25 = 42.



Question 7.

What is a composite number?


Answer:

A whole number that can be divided evenly by numbers other than 1 or itself is a composite number. We can also say, “A non - prime number is a composite number”.



Question 8.

If a and b are relatively prime then what is their HCF?


Answer:

Two relatively prime numbers don’t have a common factor other than 1. So HCF = 1.



Question 9.

If the rational number a/b has a terminating decimal expansion, what is the condition to be satisfied by b?


Answer:

b = (2m× 5m), where m and n are some non - negative integers.



Question 10.

Simplify: .


Answer:

2√45 = 2√(3*3*5)
= 2*3√5
=6√5

Also 3√20 = 3√2*2*5
= 3*2√5
= 6√5

So numerator = 6√5 + 6√5 = 12√5

And denominator is 2√5.

∴ expression becomes = 6


Question 11.

Write the decimal expansion of .


Answer:

.



Question 12.

Show that there is no value of n for which (2” × 5”) ends in 5.


Answer:

(2” × 5”) = 10n {using rules of exponent}

And 10n either ends up with 1 for n = 0 and for n>0 and integer, ends up with zero.


Also for negative value of n it ends with fractions not ending with 5.


∴ this will never end with 5.



Question 13.

Is it possible to have two numbers whose HCF is 25 and LCM is 520?


Answer:

No, ∵ HCF does not divided LCM exactly

Using Euclid’s division lemma -


Take a = 520 and b = 25.


a = bq + r. where q is the quotient, r is the remainder and b is the divisor.


If HCF divides LCM completely, r = 0.


Here 520 = 25(20) + 20


So, r = 20


∵ r is not equal to zero


∴ HCF does not divides LCM completely.


So this is not possible for two numbers to have HCF = 25 and LCM = 520.



Question 14.

Give an example of two irrationals whose sum is rational.


Answer:

Let’s take (2 + ) and (2 ‒ ).

These are irrational numbers.


Their sum = (2 + ) + (2 ‒ ) = 4 {which is rational}.



Question 15.

Give an example of two irrationals whose product is rational.


Answer:

Let’s take (3 + ) and (3 ‒ ).

These are irrational numbers.


Their product = (3 + ) × (3 ‒ ) = 9 + 3√2 – 3√2 – (√2)2


= 9 – 4 = 5 {which is rational}



Question 16.

If a and b are relatively prime, what is their LCM?


Answer:

Two relatively prime numbers don’t have a common factor other than 1.

i.e, HCF = 1 and we know HCF × LCM = product of numbers.


So LCM = product of numbers = a × b = ab.



Question 17.

The LCM of two numbers is 1200. Show that the HCF of these numbers cannot be 500. Why?


Answer:

No, ∵ HCF should divide LCM exactly.

Using Euclid’s division lemma -


Take a = 1200 and b = 500.


a = bq + r. where q is the quotient, r is the remainder and b is the divisor.


If HCF divides LCM completely, r = 0.


Here 1200 = 500(2) + 200


r = 200


∵ r is not equal to zero.


∴ HCF does not divides LCM completely.


So this is not possible for two numbers to have HCF = 500 and LCM = 1200.



Question 18.

Express as a rational number in simplest form.


Answer:

Let x = 0. - (i)

Multiply 10 on both sides -


10x = 4. - (ii)


Subtract the equations (i) from (ii) -


⇒ 9x = 4


So x =



Question 19.

Express as a rational number in simplest form.


Answer:

Let x = 0. - (i)

Multiply 100 on both sides -


100x = 23. - (ii)


Subtract the equations (i) from (ii) -


⇒ 99x = 23


So x =



Question 20.

Express why 0.15015001500015… is an irrational number.


Answer:

Irrational numbers are non - terminating non - recurring decimals.

Thus, 0.15015001500015 … is an irrational number.



Question 21.

Show that is irrational.


Answer:

assume that is rational
= a/b, where a and b are integers.
⇒ √2 = 3a/b
we know that a, b and 3 are integers and they are also rational {i.e RHS is rational}
therefore √2 will be rational.
but we know that √2 is irrational.
there is a contradiction
so, is an irrational number.



Question 22.

Write a rational number between √3 and 2.


Answer:

∵, √3 = 1.732…

So, we may take 1.8 as the required rational number between √3 and 2.


Thus, the required rational number is 1.8 or


Question 23.

Explain why is a rational number.


Answer:

∵, 3. is a non - terminating repeating decimal.

∴, is a rational number.




Multiple Choice Questions (mcq)
Question 1.

Which of the following is a pair of co - primes?
(A) (14,35) (B) (18,25) (C) (31,93) (D) (32,62)
A. (14, 25)

B. (18, 25)

C. (31, 93)

D. (32, 62)


Answer:

Co - prime numbers are the numbers which have only one common divisor = 1 or their HCF = 1.

Here for option


(A) 14 = 2 × 7
35 = 5 × 7

∴HCF(14,25) = 7.


(B)18 = 2 × 3 × 3
25 =5 × 5


∴ HCF(18,25) = 1.


(C) 31 = 1 × 31

93 = 3 × 31


∴ HCF(31,93) = 31.


(D) 32 = 2 × 2 × 2 × 2 × 2

62 = 2 × 31

∴ HCF(32,62) = 2.


So answers will be (B).


Question 2.

If a = (22× 33× 54) and b = (23× 32× 5) then HCF (a, b) = ?
A. 90

B. 180

C. 360

D. 540


Answer:

a = (22× 33× 54)


b = (23× 32× 5)


Here HCF (a, b) = product of common factors = 22 × 32 × 5 = 4 × 9 × 5 = 180.


Question 3.

HCF of (23× 32× 5), (22× 33× 52) and (24× 3 × 53× 7) is
A. 30

B. 48

C. 60

D. 105


Answer:

HCF is product of common factors,

Here HCF = 22 × 3 × 5 = 4 × 3 × 5 = 60.


Question 4.

LCM of (23× 3 × 5) and (24× 5 × 7) is
A. 40

B. 560

C. 1120

D. 1680


Answer:

LCM of the given numbers = product of prime factors with highest powers = 24× 5 × 7 × 3 = 1680.


Question 5.

The HCF of two numbers is 27 and their LCM is 162. If one of the numbers is 54, what is the other number?
A. 36

B. 45

C. 9

D. 81


Answer:

We know that product of numbers = LCM × HCF.

Let the other number be x.


So, x × 54 = 27 × 162


Or x = 81.


Question 6.

The product of two numbers is 1600 and their HCF is 5. The LCM of the numbers is
A. 8000

B. 1600

C. 320

D. 1605


Answer:

We know that product of numbers = LCM × HCF

So, 1600 = 5 × LCM


And ∴ LCM = 320.


Question 7.

What is the largest number that divides each one of 1152 and 1664 exactly?
A. 32

B. 64

C. 128

D. 256


Answer:

The largest number that can divide both the numbers will be the HCF of the two numbers -

So prime factors of the numbers –


1152 = 27 × 32


1664 = 27 × 13.


∴ HCF of number = 27 = 128.


Question 8.

What is the largest number that divides 70 and 125, leaving remainders 5 and 8 respectively?
A. 13

B. 9

C. 3

D. 585


Answer:

∵ we are getting 5 and 8 as remainder.

Subtracting 5 and 8 from the dividends will give us the exact division.


So let’s subtract the remainder and find the HCF of numbers (largest number which divides both the number) -


We need to find HCF of 70 - 5 = 65 and 125 - 8 = 117.


Prime factorization of numbers -


65 = 5 × 13


117 = 3 × 3 × 13


HCF of given numbers = 13.


Question 9.

What is the largest number that divides 245 and 1029, leaving remainder 5 in each case?
A. 15

B. 16

C. 9

D. 5


Answer:

∵ we are getting 5 as remainder.

Subtracting 5 from the dividend will give us the exact division.


So let’s subtract the remainder and find the HCF of numbers (largest number which divides both the number) -


We need to find HCF of 245 - 5 = 240 and 1029 - 5 = 1024.


Prime factorization of numbers -


240 = 2 × 2 × 2 × 2 × 3 × 5


1024 = 2 × 2 × 2 × 2 × 2 × 2 × 2 × 2 × 2 × 2


HCF of given numbers = 2 × 2 × 2 × 2 = 16.


Question 10.

The simplest form of is
A.

B.

C.

D.


Answer:

Prime factors of the numbers –

1095 = 3 × 5 × 73


1168 = 24 × 73


Dividing them -


We get numerator as 3 × 5 = 15 and denominator as 24 = 16.


Question 11.

Euclid’s division lemma states that for any positive integers a and b, there exist unique integers q and r such that a = bq + r, where r must satisfy
A. 1 < r < b

B. 0 < r ≤ b

C. 0 ≤ r < b

D. 0 < r < b


Answer:

The theorem itself.


Question 12.

A number when divided by 143 leaves 31 as remainder. What will be the remainder when the same number is divided by 13?

(a) 0
(b) 1
(c) 3
(d) 5
A. 0

B. 1

C. 3

D. 5


Answer:

Given: A number when divided by 143 leaves 31 as remainder.

To find: the remainder when the same number is divided by 13.

Solution:

Let the number be a.

By Euclid’s division lemma -

Putting the values of divisor as 143 and remainder as 31.

a = 143(q) + 31, where q is the quotient when divided by 143.

a = {13(11)}q + 31

a = 13(11)q + 13(2) + 5

a = 13(11q + 2) + 5 ..... (i)


Again when it is divided by 13, let the remainder be r.


So, a = 13(p) + r ......(ii)

where p is the quotient when divided by 13.


Comparing (i) and (ii) -


11q + 2 = p and r = 5.


So remainder = 5.


Question 13.

Which of the following is an irrational number?
A.

B. 3.1416

C.

D. 3.141141114…


Answer:

An irrational number is non - terminating non-repeating decimal and can’t be expressed in p/q form. So here option D is neither terminating non-repeating decimal.


Question 14.

π is
A. an integer

B. a rational number

C. an irrational number

D. none of these


Answer:

∵ π is non - terminating non-periodic in nature and does not satisfy the rational number definition. It is irrational.


Question 15.

is
A. an integer

B. a rational number

C. an irrational number

D. none of these


Answer:

∵ the given number is non - terminating but repeating decimal. ∴ it’s a rational number.


Question 16.

2.13113111311113… is
A. an integer

B. a rational number

C. an irrational number

D. none of these


Answer:

∵ the given number is neither terminating nor repeating decimal, it’s an irrational number.


Question 17.

The number 3.24636363… is
A. an integer

B. a rational number

C. an irrational number

D. none of these


Answer:

∵ the number is non - terminating but repeating decimal as after 2 places of decimal it repeats. ∴ it’s a rational number.


Question 18.

Which of the following rational numbers is expressible as a terminating decimal?
A.

B.

C.

D.


Answer:

For the terminating decimal the denominator must be in the form 2n × 5m where m and n are non - negative integers.

For option C denominator is 625 which can be written as 20 × 54.


∴ option C is the answer.


Question 19.

The decimal expansion of the rational number will terminate after
A. one decimal place

B. two decimal places

C. three decimal places

D. four decimal places


Answer:

Denominator = 22 × 5 = 2 × 10.

So, expression is equal to = = 1.85


∴ it terminates after two decimal places.


Question 20.

The decimal of the number will terminate after
A. one decimal place

B. two decimal places

C. three decimal places

D. four decimal places


Answer:

The expression = 11.8024, which terminates after 4 places of decimal.


Question 21.

The number 1.732 is
A. an irrational number

B. a rational number

C. an integer

D. a whole number


Answer:

∵ the given number is terminating decimal.

∴ it’s a rational number.


Question 22.

a and b are two positive integers such that the least prime factor of a is 3 and the least prime factor of b is 5. Then, the least prime factor of (a + b) is
A. 2

B. 3

C. 5

D. 8


Answer:

Least prime factor being 3 or 5 means that 2 is not a factor.

So the numbers are both odd.


a + b will be even no. as a is odd and b is odd {∵ odd + odd = even}


Least prime factor of an even no is 2.


Question 23.

is
A. a rational number

B. an irrational number

C. a terminating decimal

D. a nonterminating repeating decimal


Answer:

is a non - terminating non - repeating decimal. So, it’s an irrational number.


Question 24.

is
A. a fraction

B. a rational number

C. an irrational number

D. none of these


Answer:

Assume to be rational. So we can write it in the form of a/b where a and b are co - prime.

So , a/b =


And ∴ b/a = √2


Since b/a is rational but √2 is irrational.


By contradiction is irrational.


Question 25.

is
A. an integer

B. a rational number

C. an irrational number

D. none of these


Answer:

Sum of a rational and an irrational number is an irrational number.

Here 2 is rational and is irrational. So their sum is irrational.


Question 26.

What is the least number that is divisible by all the natural numbers from 1 to 10 (both inclusive)?
A. 100

B. 1260

C. 2520

D. 5040


Answer:

Least number that is divisible by all the natural numbers from 1 to 10 will be LCM of the numbers -

LCM of numbers from 1 to 10 = product of prime factors with highest powers = 23 × 32 × 5 × 7 = 2520.



Formative Assessment (unit Test)
Question 1.

The decimal representation of is
A. a terminating decimal

B. a nonterminating, repeating decimal

C. a nonterminating and nonrepeating decimal

D. none of these


Answer:

Here the denominator 150 = 2 × 3 × 52 which is not of the form 2n × 5m.Hence the given fraction is non - terminating in nature.

Also (multiply and divide by 2)


Question 2.

Which of the following has a terminating decimal expansion?
A.

B.

C.

D.


Answer:

For a terminating decimal, the denominator must be in the form 2n × 5m where m and n are non negative integers.

For option B denominator is 80 which can be written as 24 × 5.


∴ option B is the answer.


Question 3.

One dividing a positive integer n by 9, we get 7 as remainder. What will be the remainder if (3n – 1) is divided by 9?
A. 1

B. 2

C. 3

D. 4


Answer:

Using Euclid’s division lemma –

n = 9(q) + 7, where q is the quotient when divided by 9.


Using this, we have

3n - 1 = 3(9q + 7) - 1
⇒ 3n - 1 = 27q + 21 - 1
⇒ 3n - 1 = 27q + 20
Now, we have to set the above equation in a way, such that
3n - 1 = 9q' + r', where 0 ≤ r < 9

⇒ 3n - 1 = 27q + 18 + 2
⇒ 3n - 1 = 9(3q + 2) + 2

According to Euclid's divison lemma, we have
r = 2, when 3n - 1 is divided by 9


Question 4.

Solve:


A.

B.

C.

D. None of these


Answer:

0. = and 0. = .


∴,0. + 0. =


Question 5.

Show that any number of the form 4n, n ∈ N can never end with the digit 0.


Answer:

For a number to end with zero, it must contain 5 as one of it’s prime factor.

In case of 4n, 5 can’t be a prime factor. So it can’t end with zero.



Question 6.

The HCF of two numbers is 27 and their LCM is 162. If one of the number is 81, find the other.


Answer:

Let the other number be x.

We know product of numbers = LCM × HCF.


So, x × 81 = 27 × 162


And x = 54.



Question 7.

Examine whether is a terminating decimal.


Answer:

The denominator 30 can be written as 2 × 3 × 5, which is not in the form 2n × 5m.

∴ it is not a terminating decimal.



Question 8.

First the simplest form of .


Answer:

Prime factors of the numbers are -

148 = 2 × 2 × 37


185 = 5 × 37


So HCF = 37


Dividing the numbers we get numerator as 4 and denominator as 5.



Question 9.

Which of the following number are irrational?

a. b.

c. 3.142857 d.

e. π f.

g.


Answer:

, , π, 0.232332333…


An irrational number is non - terminating, non - repeating and cannot be represented in a fractional form. So the decimals which are non - terminating and non - repeating are irrationals.



Question 10.

Prove that (2 +√3) is irrational.


Answer:

√3 is an irrational number and addition of a rational number and an irrational number is an irrational number.


∴, 2 + √3 is irrational, where √3 is an irrational number while 2 is a rational number.


Alternative - Assume that 2 + √3 is rational
, where a and b are integers.


we know that a, b and 2 are integers, therefor a - 2b is also an integer
therefore √3 will be rational.
but we know that √3 is irrational.
there is a contradiction
so, 2 + √3 is an irrational number


Question 11.

Find the HCF and LCM of 12, 15, 18, 27.


Answer:

12 = 2 × 2 × 3

15 = 3 × 5


18 = 2 × 3 × 3


27 = 3 × 3 × 3


So HCF = 3


And LCM = product of prime factors with highest powers = 22 × 33 × 5 = 540



Question 12.

Give an example of two irrationals whose sum is rational.


Answer:

Let’s take (2 + ) and (2 ‒ ).

These are irrational numbers.


Their sum = (2 + ) + (2 ‒ ) = 4 {which is rational}.



Question 13.

Give prime factorization of 4620.


Answer:

4620 = 2 × 2 × 3 × 5 × 7 × 11 = 22× 3 × 5 × 7 × 11



Question 14.

Find the HCF of 1008 and 1080 by prime factorization method.


Answer:

Prime factors of numbers are -

1008 = 2 × 2 × 2 × 2 × 3 × 3 × 7


1080 = 2 × 2 × 2 × 3 × 3 × 3 × 5


HCF = 2 × 2 × 2 × 3 × 3 = 72



Question 15.

Find the HCF and LCM of , and .


Answer:

HCF of numerators (8,10,16) = 2 and HCF of denominators (9,27,81) = 9.

LCM of numerators(8,10,16) = 80 and LCM of denominators(9,27,81) = 81.


So HCF of fraction == .


and LCM of fraction == .


Question 16.

Find the largest number which divides 546 and 764, leaving remainders 6 and 8 respectively.


Answer:

∵ we are getting 6 and 8 as remainder.

Subtracting 6 and 8 from the dividends will give us the exact division.


So let’s subtract the remainder and find the HCF of numbers (largest number which divides both the number) -


We need to find HCF of 546 - 6 = 540 and 764 - 8 = 756.


Prime factorization of numbers -


540 = 2 × 2 × 3 × 3 × 3 × 5


756 = 2 × 2 × 3 × 3 × 3 × 7


HCF of given numbers = 2 × 2 × 3 × 3 × 3 = 108



Question 17.

Prove that is an irrational number.


Answer:

Let us assume that √3 is a rational number.

That is, we can find integers a and b (≠ 0) such that √3 = (a/b)


Suppose, a and b have a common factor other than 1, then we can divide by the common factor, and assume that a and b are co - prime.


So, √3b = a


⇒ 3b2 = a2 (Squaring on both sides)


Therefore, a2 is divisible by 3.


∴ ‘a’ is also divisible by 3.


So, we can write a = 3c for some integer c.


Equation (1) becomes,


3b2 = (3c)2


3b2 = 9c2


b2 = 3c2


This means that b2 is divisible by 3, and so b is also divisible by 3.


Therefore, a and b have at least 3 as a common factor.


But this contradicts the fact that a and b are co - prime.


This contradiction has arisen because of our incorrect assumption that √3 is rational.


So, we conclude that √3 is irrational.



Question 18.

Show that every positive odd integer is of the form (4q + 1) or (4q + 3) for some integer q.


Answer:

Let a be any odd positive integer and b = 4.

By division lemma there exist integer q and r such that


a = 4 q + r, where 0 ≤ r ≤ 4


so a = 4q or, a = 4q + 1 or, a = 4q + 2 or, a = 4q + 3


4q + 1 4 is divisible by 2 but 1 is not divisible by 2,so it is an odd number


4q + 2 4 is divisible by 2 and 2 is also divisible by 2,so it is an even number


4q + 3 4 is divisible by 2 but 3 is not divisible by 2,so it is an odd number


4q + 4 4 is divisible by 2 and 4 is also divisible by 2,so it is an even number


∴, any odd integer is of the form 4q + 1 or, 4q + 3.



Question 19.

Show that one and only one out of n, (n + 2) and (n + 4) is divisible by 3, where n is any positive integer.


Answer:

We applied Euclid Division algorithm on n and 3.

a = bq + r on putting a = n and b = 3


n = 3q + r, 0 < r < 3


So,


n = 3q (I)


n = 3q + 1 (II)


n = 3q + 2 (III)


Case - I: When n = 3q


In this case, we have


n = 3q, which is divisible by 3


Now, n = 3q


n + 2 = 3q + 2


n + 2 leaves remainder 2 when divided by 3


Again, n = 3q


n + 4 = 3q + 4 = 3(q + 1) + 1


n + 4 leaves remainder 1 when divided by 3


n + 4 is not divisible by 3.


Thus, n is divisible by 3 but n + 2 and n + 4 are not divisible by 3.


Case - II: when n = 3q + 1


In this case, we have


n = 3q + 1,


n leaves remainder 1 when divided by 3.


n is divisible by 3


Now, n = 3q + 1


n + 2 = (3q + 1) + 2 = 3(q + 1)


n + 2 is divisible by 3.


Again, n = 3q + 1


n + 4 = 3q + 1 + 4 = 3q + 5 = 3(q + 1) + 2


n + 4 leaves remainder 2 when divided by 3


n + 4 is not divisible by 3.


Thus, n + 2 is divisible by 3 but n and n + 4 are not divisible by 3.


Case - III: When n + 3q + 2


In this case, we have


n = 3q + 2


n leaves remainder 2 when divided by 3.


n is not divisible by 3.


Now, n = 3q + 2


n + 2 = 3q + 2 + 2 = 3(q + 1) + 1


n + 2 leaves remainder 1 when divided by 3


n + 2 is not divisible by 3.


Again, n = 3q + 2


n + 4 = 3q + 2 + 4 = 3(q + 2)


n + 4 is divisible by 3.


Thus, n + 4 is divisible by 3 but n and n + 2 are not divisible by 3.



Question 20.

Show that is irrational.


Answer:

3√2 is an irrational number and subtraction of a rational number and an irrational number is an irrational number.

∴ 4 + 3√2 is irrational, where 3√2 is an irrational number while 4 is a rational number.


Alternative - Assume that 4 + 3√2 is rational
4 + 3√2 = a/b, where a and b are integers.
⇒ 3√2 = a/b - 4
⇒ 3√2 = a/b – 4b/b
⇒ √2 = (a – 4b) / 3b
we know that a, b, 3 and 4 are integers and they are also rational {i.e RHS is rational}
therefore √2 will be rational.
but we know that √2 is irrational.
there is a contradiction
so, 4 + 3√2 is an irrational number